Đến nội dung

conan98md nội dung

Có 99 mục bởi conan98md (Tìm giới hạn từ 27-04-2020)



Sắp theo                Sắp xếp  

#483969 $a,8x^{2}-13x+1=(1+\frac{1}{x})\...

Đã gửi bởi conan98md on 19-02-2014 - 14:30 trong Phương trình - hệ phương trình - bất phương trình



giải phương trình và hệ phương trình sau :

$a,8x^{2}-13x+1=(1+\frac{1}{x})\sqrt[3]{3x^{2}-2}$

 

$b,\sqrt{1-x^{2}}+\sqrt[4]{x^{2}+x-1}+\sqrt[6]{1-x}=1$

 

$c,x(3+2x^{2}-x^{4})=\sqrt{3}(3x^{4}+2x^{2}-1)$

 

$d,\sqrt{x^{2}-1}+x=\sqrt{x^{3}-2}$

 

$e,\left\{\begin{matrix} \frac{1}{x}-\frac{1}{2y}=2(y^{4}-x^{4})\\ \frac{1}{x}+\frac{1}{2y}=(3x^{2}+y^{2})(3y^{2}+x) \end{matrix}\right.$

 

hình như đầu bài như thế này mới đúng $e,\left\{\begin{matrix} \frac{1}{x}-\frac{1}{2y}=2(y^{4}-x^{4})(1)\\ \frac{1}{x}+\frac{1}{2y}=(3x^{2}+y^{2})(3y^{2}+x^{2}) \end{matrix}\right.$(2)

 

lấy (1)+(2) : 1=$y^{5}+5x^{4}y+10x^{2}y^{3}$

 
lấy (1)-(2) : 2=$x^{5}+5xy^{4}+10x^{3}y^{2}$
 
$\Rightarrow$  $\left\{\begin{matrix} 3=(x+y)^{5}\\ 1=(x-y)^{5} \end{matrix}\right.$

 




#481950 $\sqrt{x^{2}+15}= 3x-2+\sqrt{x^{...

Đã gửi bởi conan98md on 08-02-2014 - 15:12 trong Phương trình - hệ phương trình - bất phương trình

Giải phương trình

$\sqrt{x^{2}+15}= 3x-2+\sqrt{x^{2}-8}$

pt $\Leftrightarrow$ $(x-1)(\frac{x+1}{\sqrt{x^{2}+15}+4}-\frac{x+1}{\sqrt{x^{2}+8}+3}-3)$ =0

 
xét f(x)=$(x+1)(\frac{1}{\sqrt{x^{2}+15}+4}-\frac{1}{\sqrt{x^{2}+8}+3})$-3=0
 
 
Ta có ; $\sqrt{x^{2}+15}$ > $\sqrt{x^{2}+8}$ kết hợp với pt ban đầu $\Rightarrow$ x> $\frac{2}{3}$$\Rightarrow$ x+1> 0
 
Mà $\frac{x+1}{\sqrt{x^{2}+15}+4}-\frac{x+1}{\sqrt{x^{2}+8}+3}$< 0
 
$\Rightarrow$ f(x)< 0



#481949 Giải phương trình $ \sqrt{x^2+15}=3\sqrt[3]{x...

Đã gửi bởi conan98md on 08-02-2014 - 15:10 trong Phương trình - hệ phương trình - bất phương trình



Giải phương trình $ \sqrt{x^2+15}=3\sqrt[3]{x}-2+\sqrt{x^2+8} $

 

????????????????????????????????????




#464766 giải hệ $\left\{\begin{matrix} (x+1)^...

Đã gửi bởi conan98md on 17-11-2013 - 07:41 trong Phương trình - hệ phương trình - bất phương trình

giải hệ $\left\{\begin{matrix} (x+1)^{2}=6y-\frac{6}{y} &  & \\ (y+1)^{2}=6x-\frac{6}{x}  &  &  \end{matrix}\right.$

 




#459135 tìm min A = $(x^{3}+1)(x^{3}+1)+\frac{xy...

Đã gửi bởi conan98md on 21-10-2013 - 21:10 trong Bất đẳng thức và cực trị

cho x,x>0 thoả mãn x+y+xy=3

 

tìm min A = $(x^{3}+1)(x^{3}+1)+\frac{xy}{x+y}$




#458045 Giải phương trình: $2\sqrt[3]{3x-2}+3\sqrt{6-5x...

Đã gửi bởi conan98md on 16-10-2013 - 22:59 trong Phương trình, hệ phương trình và bất phương trình

giải pt :

 

1. $\sqrt{4x-1}+\sqrt{4x^{2}-1}$=1

 
2 .$\sqrt{x^{2}-\frac{7}{x^{2}}}+\sqrt{x-\frac{7}{x^{2}}}$=x
 
3..$2\sqrt[3]{3x-2}+3\sqrt{6-5x}$-8=0

 

 




#451594 Cho $a\geq 1;b\geq 1$.CMR:$a\sqrt{b-1...

Đã gửi bởi conan98md on 18-09-2013 - 22:48 trong Bất đẳng thức và cực trị

áp dụng BDT AM-GM:

 

$a\sqrt{b-1}$=$a\sqrt{1(b-1)}$ $\leq $ $\frac{ab}{2}$

 

CM tương tự -> đpcm




#451473 Giải phương trình: 1/ $2\sqrt{2x+4}+4\sqrt{2-...

Đã gửi bởi conan98md on 18-09-2013 - 17:56 trong Phương trình - hệ phương trình - bất phương trình



Giải phương trình:

1/   $2\sqrt{2x+4}+4\sqrt{2-x}=\sqrt{9x^{2}+16}$

2/   $(4x-1)\sqrt[3]{2-8x^{3}}=2x$

3/   $x(4x^{2}+1)+(x-3)\sqrt{5-2x}=0$

2/ pt $\Leftrightarrow $ $\sqrt[3]{2-8x^{3}}$ = $\frac{2x}{4x-1}$
 
$\Leftrightarrow $ $\frac{1-8x^{3}}{\sqrt[3]{(2-8x^{3})^{2}}+\sqrt[3]{2-8x^{3}}+1}$ = $\frac{-2x+1}{4x-1}$



#451466 $(4x-1)(\sqrt{x+3}+\sqrt[3]{3x+5})=4x+8$

Đã gửi bởi conan98md on 18-09-2013 - 17:43 trong Phương trình - hệ phương trình - bất phương trình



Giải phương trình: $(4x-1)(\sqrt{x+3}+\sqrt[3]{3x+5})=4x+8$ 

                                                     (Đề thi chọn HSG trường mình)

pt $\Leftrightarrow$ $\sqrt{x+3} +\sqrt[3]{3x-5}$ = $\frac{4x+8}{4x-1}$
$\Leftrightarrow $ $\frac{x-1}{\sqrt{x+3}+2}+\frac{3(x-1)}{\sqrt[3]{(3x+5)^{2}}+2\sqrt[3]{3x+5}+4}$ = $\frac{-12(x-1)}{4x-1}$



#450501 $\left\{\begin{matrix} \frac{1...

Đã gửi bởi conan98md on 15-09-2013 - 09:11 trong Phương trình - hệ phương trình - bất phương trình



Giải hệ phương trình:

1) $\left\{\begin{matrix} \sqrt[4]{x}(\frac{1}{4}+\frac{2\sqrt{x}+\sqrt{y}}{x+y})=2\\ \sqrt[4]{y}(\frac{1}{4}-\frac{2\sqrt{x}+\sqrt{y}}{x+y})=1 \end{matrix}\right.$

2) $\left\{\begin{matrix} \frac{1}{x}+\frac{1}{2y}=(x^2+3y^2)(3x^2+y^2)\\\frac{1}{x}-\frac{1}{2y}=2(y^4-x^4) \end{matrix}\right.$

bài 2: 

 

cộng 2 pt ta có :$\frac{2}{x}$=$(3x^{2}+y^{2})(x^{2}+3y^{2})+2(y^{4}-x^{4})$

 
$\Rightarrow$ 2= $x^{5}+10x^{3}y^{2} +5xy^{4}$ (*)
 
trừ hai pt ta có :$\frac{1}{y}$ = $(3x^{2}+y^{2})(x^{2}+3y^{2})-2(y^{4}-x^{4})$
 
$\Rightarrow$ 1 =$5x^{4}y+10x^{2}y^{3} +y^{5}$ (**)
 
cộng (*) và (**) ta có : 3=$(x+y)^{5}$
 
trừ (*) và (**) ta có 1=$(x-y)^{5}$
 
$\rightarrow $ (x;y)= $(\frac{\sqrt[5]{3}+1}{2};\frac{\sqrt[5]{3}-1}{2})$

 




#450200 Bất đẳng thức chuẩn bị cho kì thi THPTQG 2015-2016

Đã gửi bởi conan98md on 14-09-2013 - 15:33 trong Bất đẳng thức và cực trị

 

Bài 5: 

Cho $x,y,z$ là nghiệm của hệ 2 phương trình:

$x^2  + xy + y^2  = 3 $ và $y^2  + yz + z^2  = 16 $

 
Tìm max của: $P=xy + yz + zx$

 

từ gt $\Rightarrow$ 48=$(x^{2}+xy+y^{2})(y^{2}+yz+z^{2})$
 
=$((x+\frac{y}{2})^{2}+\frac{3x}{4})$$(\frac{3z}{4}+(y+\frac{z}{2})^{2})$
 
áp dụng BDT Cauchy-Schwarz
 
$\Rightarrow$ 48 $\geq$ $(\frac{\sqrt{3}z}{2}(\frac{x}{2}+y)+\frac{\sqrt{3}x}{2}(y+\frac{z}{2}))^{2}$
 
$\Rightarrow$ 48 $\geq$ $\frac{3}{4}(xy+yz+xz)^{2}$
 
$\Rightarrow$  P $\leq$  8   



#450196 $\left\{\begin{matrix} \sqrt{x...

Đã gửi bởi conan98md on 14-09-2013 - 14:58 trong Phương trình - hệ phương trình - bất phương trình

Giải hệ phương trình:;

$\left\{\begin{matrix} \sqrt{x}-\sqrt{x-y-1}=1\\y^2+x+2y\sqrt{x}-y^2x=0 \end{matrix}\right.$

 

đk : x $\geq$ 0; x-y-1$\geq$0

 

(1) <-> $\sqrt{x}$=$\sqrt{x-y-1}$+1 $\Leftrightarrow$ x=x-y+2$\sqrt{x-y-1}$

 
$\Leftrightarrow$ y=2$\sqrt{x-y-1}$
 
$\Leftrightarrow$ $y^{2}$= 4(x-y-1)
 
$\Leftrightarrow$ $(y+2)^{2}$=4x
 
$\Leftrightarrow$ y+2 = 2$\sqrt{x}$ (*)
 
(2)$\Leftrightarrow$ $(y+\sqrt{x})^{2}$ = $xy^{2}$
 
$\Leftrightarrow$ y+$\sqrt{x}$ = y$\sqrt{x}$ (**)
 
từ (*) và (**) $\Rightarrow $ (x;y) =  (4;2)



#447789 $\begin{cases} a^3 +b^3 = 1 \\ a^5+b^5 =1...

Đã gửi bởi conan98md on 04-09-2013 - 17:10 trong Phương trình, hệ phương trình và bất phương trình

bài 2

 

từ pt (2) $\Rightarrow$ $x^{4}$ $\leq$ 1 $\Rightarrow$ -1$\leq$ x ,y $\leq$ 1

 

 

vì x $\leq$ 1 $\Rightarrow$ $x^{3}$ $\leq$ 1 do đó theo (1) ta có :y$\geq$0

 

vì y $\leq$ 1 $\Rightarrow$ $y^{3}$ $\leq$ 1 do đó theo (1) ta có :x$\geq$0

 

 $\Rightarrow$ 0 $\leq$ x,y $\leq$ 1

 

từ (1) (2) $\Rightarrow$ $x^{3}$(1-x) + $y^{3}$(1-y) =0

 

$\Rightarrow$ x=0;y=1 hoặc x=1;y=0

 

 

 

 

 

 

 




#447256 $\sum \frac{a+b}{ab+c^{2}} \leq \sum \frac{1}{a...

Đã gửi bởi conan98md on 02-09-2013 - 15:13 trong Bất đẳng thức và cực trị

CM rằng :$\frac{a+b}{ab+c^{2}}+\frac{b+c}{bc+a^{2}}+\frac{a+c}{ca+b^{2}}$ $\leq $ $\frac{1}{a}+\frac{1}{b}+\frac{1}{c}$

 




#446765 $(3+\frac{1}{b}+\frac{1}{b...

Đã gửi bởi conan98md on 01-09-2013 - 10:04 trong Bất đẳng thức và cực trị

1/ P= $(3+\frac{1}{b}+\frac{1}{b})(3+\frac{1}{b}+\frac{1}{c})(3+\frac{1}{c}+\frac{1}{a})$. tìm min P

 

Trong đó: a,b,c > 0 thoả mãn a+b+c $\leq $  $\frac{3}{2}$

 

2/ CM : $\frac{a}{b+c+1}+\frac{b}{a+c+1}+ \frac{c}{a+b+1}$+(1-a)(1-b)(1-c) $\leq$ 1

 

 
với 0 $\leq$ a,b,c $\leq$ 1

 

 




#439456 CM: N là trung điểm của AC

Đã gửi bởi conan98md on 31-07-2013 - 10:38 trong Hình học

Cho Δ ABC cân tại A nội tiếp (O) . từ C kẻ đường thẳng // với AB cắt (O) tại D ; AD cắt BC tại M; MO

 

cắt AC tại E ; TỪ E kẻ đường thẳng // với MB cắt CD tại F ; MF cắt AC tại N ; I là trung điểm của EF

 

 

1/ N là trung điểm của AC

 

2/ IN // AM

 

 

 

 




#439427 cm:(1+ $\frac{1}{a^{2}}$)( 1+...

Đã gửi bởi conan98md on 31-07-2013 - 00:01 trong Bất đẳng thức và cực trị

cho a,b,c >0 thoả mãn a+b+c=3

 

tìm min :(1+ $\frac{1}{a^{2}}$)( 1+ $\frac{1}{b^{2}}$)( 1 + $\frac{1}{c^{2}}$) 




#438933 giải pt nghiệm nguyên

Đã gửi bởi conan98md on 28-07-2013 - 18:41 trong Phương trình, hệ phương trình và bất phương trình

1/  x2 + y+ x + y =8

 

 

2/  x2 + y= x + y +xy

 




#436512 CM rằng : $\frac{a+1}{1+b^{2}}$...

Đã gửi bởi conan98md on 20-07-2013 - 15:44 trong Bất đẳng thức và cực trị

cho a,b,c là ba số thực dương thoả mãn : a+b+c = 3

 

CM rằng : $\frac{a+1}{1+b^{2}}$ + $\frac{b+1}{1+c^{2}}$ + $\frac{c+1}{1+a^{2}}$ $\geq$ 3 

 

 

 

 




#433447 CM: E nằm trên một cung tròn cố định khi M chạy trên BC

Đã gửi bởi conan98md on 07-07-2013 - 10:00 trong Hình học

a, ta có BD=BM.DN

 

$\Rightarrow$ Δ BDM ~ Δ NDA (c.g.c)

 

$\Rightarrow$ $\angle BDE$ = $\angle BND$

 

trong  Δ DNE có :$\angle DEN$+$\angle EDN$+$\angle END$ = 180

 

$\Rightarrow$ $\angle BED$ = 60

 

mà $\angle BCD$ =60

 

$\Rightarrow$ đpcm

 

 

b, E thuộc cung tròn chứa góc 60 dựng trên đoạn AC




#430407 $\frac{a}{a^{2}+2b+c} + \frac...

Đã gửi bởi conan98md on 25-06-2013 - 08:36 trong Bất đẳng thức và cực trị

đã có ở đây http://diendantoanho...22b3leq-frac12/




#430147 CM :$x^{2}$ + $y^{2}$ = 1

Đã gửi bởi conan98md on 24-06-2013 - 08:56 trong Đại số

bài 1 :cho x,y >0 và $x^{3}$ + $y^{3}$ = x-y . CM :$x^{2}$ + $y^{2}$ = 1

 

 

Bài 2 : tìm x,y nguyên dương để A = $\frac{x^{2}+x+1}{xy-1}$

 

bài 3 : tồn tại hay không các số a,b,c,d hữu tỷ sao cho

 

$(a+b\sqrt{2})^{1994}$ + $(c+d\sqrt{2})^{1994}$ = 5+$4\sqrt{2}$




#429543 CM : AC<BD

Đã gửi bởi conan98md on 21-06-2013 - 16:47 trong Hình học

kẻ vuông góc xuồn rồi chứng minh

KẺ vuông góc từ đỉnh nào ạ . anh nói cụ thể giúp e




#429525 CM : AC<BD

Đã gửi bởi conan98md on 21-06-2013 - 15:47 trong Hình học

cho tứ giác ABCD có $\angle BAD$ và $\angle BCD$ là góc tù . CM : AC<BD

 

 




#429011 CM :nếu a2 + b2 > 5c2 thì c là độ dài cạnh nhỏ nhất

Đã gửi bởi conan98md on 19-06-2013 - 21:24 trong Đại số

cho a,b,c là độ dài 3 cạnh của 1 tam giác .CM rằng nếu a+ b> 5c2 thì c là độ dài cạnh nhỏ nhất